Muito fácil pra ser de IMO...

2018-03-26 6:58 GMT-03:00 Anderson Torres <torres.anderson...@gmail.com>:

> Este não é o problema de alguma IMO não? Eu lembro de ter resolvido,
> quase igual à solução oficial: substituir s,t,u por a+1,b+1,c+1 e
> calcular os possiveis valores de
> 1/a+1/b+1/c + 1/ab+1/ac+1/bc usando desigualdades - para daí limitar
> os valores de a,b,c.
>
> Em 23 de março de 2018 17:01, Claudio Buffara
> <claudio.buff...@gmail.com> escreveu:
> > Enfim, nesse meio tempo acho que resolvi o problema...
> >
> > Devemos achar inteiros s, t, u, com 1 < s < t < u e tais que:
> > (stu -1)/((s-1)(t-1)(u-1)) = k  (k inteiro positivo)
> >
> > Após diversas aplicações do truque (método?) de somar e subtrair a mesma
> > coisa, chegamos a:
> > stu - 1 =  (s-1)(t-1)(u-1) + (s-1)(t-1) + (s-1)(u-1) + (t-1)(u-1) +
> (s-1) +
> > (t-1) + (u-1)
> >
> > Dividindo isso por (s-1)(t-1)(u-1), obtemos:
> > 1 + 1/(u-1) + 1/(t-1) + 1/(s-1) + 1/((t-1)(u-1)) + 1/((s-1)(u-1)) +
> > 1/((s-1)(t-1)) = k ==>
> >
> > 1/(u-1) + 1/(t-1) + 1/(s-1) + 1/((t-1)(u-1)) + 1/((s-1)(u-1)) +
> > 1/((s-1)(t-1)) = k-1
> >
> > Agora a ideia é achar cotas para s e para k.
> >
> > 1 < s < t < u ==> s >= 2, t >= 3 e u >= 4 ==> o lado esquerdo é menor ou
> > igual que:
> > 1/3 + 1/2 + 1 + 1/6 + 1/3 + 1/2 = 2+5/6
> >
> > Ou seja, como o lado esquerdo é inteiro (e positivo), só poderá ser
> igual a
> > 1 ou a 2 ==> k = 2 ou k = 3.
> >
> > Se s >= 4, então t >= 5 e u >= 6, e o lado esquerdo será, no máximo,
> igual
> > a:
> > 1/5 + 1/4 + 1/3 + 1/20 + 1/15 + 1/12 < 1.
> >
> > Logo, devemos ter s = 2 ou s = 3.
> >
> > s = 2 ==>
> > 1/(u-1) + 1/(t-1) + 1 + 1/((t-1)(u-1)) + 1/(u-1) + 1/(t-1) = k-1 ==>
> > 2/(t-1) + 2/(u-1) + 1/((t-1)(u-1)) = k-2 ==>
> > Como k-2 deve ser inteiro positivo, k só pode ser 3 e, portanto:
> > 2/(t-1) + 2/(u-1) + 1/((t-1)(u-1)) = 1 ==>
> > (2 + 1/(t-1))/(u-1) = 1 - 2/(t-1) ==>
> > u = 1 + (2t - 1)/(t - 3) = 3 + 5/(t-3) ==>
> > t = 4 e u = 8   ou   t = 8 e u = 4 (não serve pois t deve ser menor do
> que
> > u)
> >
> > s = 3 ==>
> > 1/(u-1) + 1/(t-1) + 1/2 + 1/((t-1)(u-1)) + 1/(2(u-1)) + 1/(2(t-1)) = k-1
> ==>
> > (3/2)/(u-1) + (3/2)/(t-1) + 1/((t-1)(u-1)) = k - 3/2 ==>
> > 3/(u-1) + 3/(t-1) + 2/((t-1)(u-1)) = 2k - 3 ==>
> > (3 + 2/(t-1))/(u-1) = 2k - 3t/(t-1) ==>
> > (3t - 1)/(u-1) = 2k(t-1) - 3t ==>
> > u = 1 + (3t - 1)/((2k-3)t - 2k)
> >
> > k = 2 ==> u = 1 + (3t-1)/(t-4) = 4 + 11/(t-4) ==> t = 5 e u = 15
> >
> > k = 3 ==> u = 1 + (3t-1)/(3t-6) = 2 + 5/(3t-6) ==> XXX
> >
> > As únicas soluções são:
> > (2,4,8) e (3,5,15)
> >
> > []s,
> > Claudio.
> >
> > 2018-03-23 15:38 GMT-03:00 Pedro José <petroc...@gmail.com>:
> >>
> >> Boa tarde!
> >>
> >> Aproveitando que deu o que falar o problema postado pelo Douglas, tem um
> >> que achei mais interessante.
> >>
> >> (s-1)(t-1).(u-1) | stu -1, com s, t, u inteiros  e 1 <s<t<u
> >>
> >> Saudações,
> >> Pedro
> >>
> >> --
> >> Esta mensagem foi verificada pelo sistema de antivírus e
> >> acredita-se estar livre de perigo.
> >
> >
> >
> > --
> > Esta mensagem foi verificada pelo sistema de antivírus e
> > acredita-se estar livre de perigo.
>
> --
> Esta mensagem foi verificada pelo sistema de antivírus e
>  acredita-se estar livre de perigo.
>
>
> =========================================================================
> Instru�ões para entrar na lista, sair da lista e usar a lista em
> http://www.mat.puc-rio.br/~obmlistas/obm-l.html
> =========================================================================
>

-- 
Esta mensagem foi verificada pelo sistema de antiv�rus e
 acredita-se estar livre de perigo.

Responder a